Difference between revisions of "2011 AMC 10B Problems/Problem 1"

Line 1: Line 1:
 +
== Problem 1 ==
 +
 +
What is <cmath>\dfrac{2+4+6}{1+3+5} - \dfrac{1+3+5}{2+4+6} ?</cmath>
 +
 +
<math> \textbf{(A)}\ -1\qquad\textbf{(B)}\ \frac{5}{36}\qquad\textbf{(C)}\ \frac{7}{12}\qquad\textbf{(D)}\ \frac{147}{60}\qquad\textbf{(E)}\ \frac{43}{3} </math>
 +
 +
== Solution ==
 +
 
First, simplify the fractions.
 
First, simplify the fractions.
  
 
<math>\dfrac{2+4+6}{1+3+5} - \dfrac{1+3+5}{2+4+6} = \dfrac{12}{9} - \dfrac{9}{12}</math>
 
<math>\dfrac{2+4+6}{1+3+5} - \dfrac{1+3+5}{2+4+6} = \dfrac{12}{9} - \dfrac{9}{12}</math>
  
<math>\dfrac{12}{9} - \dfrac{9}{12} = \dfrac{48}{36} - \dfrac{27}{36} = \dfrac{21}{36} = \dfrac{7}{12}</math>
+
<math>\dfrac{12}{9} - \dfrac{9}{12} = \dfrac{48}{36} - \dfrac{27}{36} = \dfrac{21}{36} = \boxed{\dfrac{7}{12} \textbf{(C)}}</math>
 
 
<math>(C) \dfrac{7}{12}</math>
 

Revision as of 18:53, 25 May 2011

Problem 1

What is \[\dfrac{2+4+6}{1+3+5} - \dfrac{1+3+5}{2+4+6} ?\]

$\textbf{(A)}\ -1\qquad\textbf{(B)}\ \frac{5}{36}\qquad\textbf{(C)}\ \frac{7}{12}\qquad\textbf{(D)}\ \frac{147}{60}\qquad\textbf{(E)}\ \frac{43}{3}$

Solution

First, simplify the fractions.

$\dfrac{2+4+6}{1+3+5} - \dfrac{1+3+5}{2+4+6} = \dfrac{12}{9} - \dfrac{9}{12}$

$\dfrac{12}{9} - \dfrac{9}{12} = \dfrac{48}{36} - \dfrac{27}{36} = \dfrac{21}{36} = \boxed{\dfrac{7}{12} \textbf{(C)}}$